K
Khách

Hãy nhập câu hỏi của bạn vào đây, nếu là tài khoản VIP, bạn sẽ được ưu tiên trả lời.

15 tháng 10 2021

1.

\(2a^2b^2+2b^2c^2+2c^2a^2-a^4-b^4-c^4>0\\ \Leftrightarrow a^4+b^4+c^4-2a^2b^2-2b^2c^2-2c^2a^2< 0\\ \Leftrightarrow\left(a^4+b^4+c^4+2a^2b^2-2b^2c^2-2c^2a^2\right)-4a^2b^2< 0\\ \Leftrightarrow\left(a^2+b^2-c^2\right)^2-4a^2b^2< 0\\ \Leftrightarrow\left(a^2+b^2-c^2-2ab\right)\left(a^2+b^2-c^2+2ab\right)< 0\\ \Leftrightarrow\left[\left(a-b\right)^2-c^2\right]\left[\left(a+b\right)^2-c^2\right]< 0\\ \Leftrightarrow\left(a-b+c\right)\left(a-b-c\right)\left(a+b-c\right)\left(a+b+c\right)< 0\left(1\right)\)

Vì a,b,c là độ dài 3 cạnh của 1 tg nên \(\left\{{}\begin{matrix}a+c>b\\a-b< c\\a+b>c\\a+b+c>0\end{matrix}\right.\Leftrightarrow\left\{{}\begin{matrix}a-b+c>0\\a-b-c< 0\\a+b-c>0\\a+b+c>0\end{matrix}\right.\)

Do đó \(\left(1\right)\) luôn đúng (do 3 dương nhân 1 âm ra âm)

Từ đó ta được đpcm

 

 

 

15 tháng 10 2021

uầy e đọc chả hỉu j lun :(

Giúp mình với! Mình đang cần gấp. Các bạn làm được bài nào thì giúp đỡ mình nhé! Cảm ơn!Bài 1: Cho các số thực dương a,b,c. Chứng minh rằng:\(\frac{a^2}{\sqrt{\left(2a^2+b^2\right)\left(2a^2+c^2\right)}}+\frac{b^2}{\sqrt{\left(2b^2+c^2\right)\left(2b^2+a^2\right)}}+\frac{c^2}{\sqrt{\left(2c^2+a^2\right)\left(2c^2+b^2\right)}}\le1\).Bài 2: Cho các số thực dương a,b,c,d. Chứng minh...
Đọc tiếp

Giúp mình với! Mình đang cần gấp. Các bạn làm được bài nào thì giúp đỡ mình nhé! Cảm ơn!

Bài 1: Cho các số thực dương a,b,c. Chứng minh rằng:

\(\frac{a^2}{\sqrt{\left(2a^2+b^2\right)\left(2a^2+c^2\right)}}+\frac{b^2}{\sqrt{\left(2b^2+c^2\right)\left(2b^2+a^2\right)}}+\frac{c^2}{\sqrt{\left(2c^2+a^2\right)\left(2c^2+b^2\right)}}\le1\).

Bài 2: Cho các số thực dương a,b,c,d. Chứng minh rằng:

\(\frac{a-b}{a+2b+c}+\frac{b-c}{b+2c+d}+\frac{c-d}{c+2d+a}+\frac{d-a}{d+2a+b}\ge0\).

Bài 3: Cho các số thực dương a,b,c. Chứng minh rằng:

\(\frac{\sqrt{b+c}}{a}+\frac{\sqrt{c+a}}{b}+\frac{\sqrt{a+b}}{c}\ge\frac{4\left(a+b+c\right)}{\sqrt{\left(a+b\right)\left(b+c\right)\left(c+a\right)}}\).

Bài 4:Cho a,b,c>0, a+b+c=3. Chứng minh rằng: 

a)\(\frac{a^3}{a^2+ab+b^2}+\frac{b^3}{b^2+bc+c^2}+\frac{c^3}{c^2+ca+a^2}\ge1\).

b)\(\frac{a^3}{a^2+b^2}+\frac{b^3}{b^2+c^2}+\frac{c^3}{c^2+a^2}\ge\frac{3}{2}\).

c)\(\frac{a+1}{b^2+1}+\frac{b+1}{c^2+1}+\frac{c+1}{a^2+1}\ge3\).

Bài 5: Cho a,b,c >0. Chứng minh rằng:

\(\frac{2a^2+ab}{\left(b+c+\sqrt{ca}\right)^2}+\frac{2b^2+bc}{\left(c+a+\sqrt{ab}\right)^2}+\frac{2c^2+ca}{\left(a+b+\sqrt{bc}\right)^2}\ge1\).

8
21 tháng 10 2019

1) Áp dụng bunhiacopxki ta được \(\sqrt{\left(2a^2+b^2\right)\left(2a^2+c^2\right)}\ge\sqrt{\left(2a^2+bc\right)^2}=2a^2+bc\), tương tự với các mẫu ta được vế trái \(\le\frac{a^2}{2a^2+bc}+\frac{b^2}{2b^2+ac}+\frac{c^2}{2c^2+ab}\le1< =>\)\(1-\frac{bc}{2a^2+bc}+1-\frac{ac}{2b^2+ac}+1-\frac{ab}{2c^2+ab}\le2< =>\)

\(\frac{bc}{2a^2+bc}+\frac{ac}{2b^2+ac}+\frac{ab}{2c^2+ab}\ge1\)<=> \(\frac{b^2c^2}{2a^2bc+b^2c^2}+\frac{a^2c^2}{2b^2ac+a^2c^2}+\frac{a^2b^2}{2c^2ab+a^2b^2}\ge1\)  (1) 

áp dụng (x2 +y2 +z2)(m2+n2+p2\(\ge\left(xm+yn+zp\right)^2\)

(2a2bc +b2c2 + 2b2ac+a2c2 + 2c2ab+a2b2). VT\(\ge\left(bc+ca+ab\right)^2\)   <=> (ab+bc+ca)2. VT \(\ge\left(ab+bc+ca\right)^2< =>VT\ge1\)  ( vậy (1) đúng)

dấu '=' khi a=b=c

21 tháng 10 2019

4b, \(\frac{a^3}{a^2+b^2}+\frac{b^3}{b^2+c^2}+\frac{c^3}{c^2+a^2}=1-\frac{ab^2}{a^2+b^2}+1-\frac{bc^2}{b^2+c^2}+1-\frac{ca^2}{a^2+c^2}\)

\(\ge3-\frac{ab^2}{2ab}-\frac{bc^2}{2bc}-\frac{ca^2}{2ac}=3-\frac{\left(a+b+c\right)}{2}=\frac{3}{2}\)

9 tháng 10 2017

Ta có \(a^2b^2+b^2c^2+c^2a^2\geq a^2b^2c^2\Leftrightarrow \frac{1}{a^2}+\frac{1}{b^2}+\frac{1}{c^2}\geq 1\)

BĐT cần chứng minh tương đương với \(\frac{\frac{1}{c^3}}{\frac{1}{a^2}+\frac{1}{b^2}}+\frac{\frac{1}{b^3}}{\frac{1}{a^2}+\frac{1}{c^2}}+\frac{\frac{1}{a^3}}{\frac{1}{b^2}+\frac{1}{c^2}}\geq \frac{\sqrt{3}}{2}\)

Đặt \((\frac{1}{a},\frac{1}{b},\frac{1}{c})=(x,y,z)\). Bài toán trở thành: 

Cho \(x,y,z>0|x^2+y^2+z^2\geq 1\). CMR \(P=\frac{x^3}{y^2+z^2}+\frac{y^3}{z^2+x^2}+\frac{z^3}{x^2+y^2}\geq \frac{\sqrt{3}}{2}\)

Lời giải:

 Áp dụng BĐT Cauchy -Schwarz:

\(P=\frac{x^4}{xy^2+xz^2}+\frac{y^4}{yz^2+yx^2}+\frac{z^4}{zx^2+zy^2}\geq \frac{(x^2+y^2+^2)^2}{x^2(y+z)+y^2(x+z)+z^2(x+y)}\) (1)

Không mất tính tổng quát, giả sử \(x\geq y\geq z\Rightarrow x^2\geq y^2\geq z^2\) 

Và \(y+z\leq z+x\leq x+y\). Khi đó, áp dụng BĐT Chebyshev: 

\(3[x^2(y+z)+y^2(x+z)+z^2(x+y)]\leq (x^2+y^2+z^2)(y+z+x+z+x+y)\)

\(\Leftrightarrow x^2(y+z)+y^2(x+z)+z^2(x+y)\leq \frac{2(x^2+y^2+z^2)(x+y+z)}{3}\)

Theo hệ quả của BĐT Am-Gm thì: \((x+y+z)^2\leq 3(x^2+y^2+z^2)\Rightarrow x+y+z\leq \sqrt{3(x^2+y^2+z^2)}\)

\(\Rightarrow x^2(y+z)+y^2(x+z)+z^2(x+y)\leq \frac{2(x^2+y^2+z^2)\sqrt{3(x^2+y^2+z^2)}}{3}\) (2)

Từ (1),(2) suy ra \(P\geq \frac{3(x^2+y^2+z^2)^2}{2(x^2+y^2+z^2)\sqrt{3(x^2+y^2+z^2)}}=\frac{\sqrt{3(x^2+y^2+z^2)}}{2}\geq \frac{\sqrt{3}}{2}\)

Ta có đpcm

Dáu bằng xảy ra khi \(x=y=z=\frac{1}{\sqrt{3}}\Leftrightarrow a=b=c=\sqrt{3}\)

5 tháng 5 2020

Đặt \(x=\frac{1}{a};y=\frac{1}{b};z=\frac{1}{c}\)

Khi đó giả thiết được viết lại là \(x^2+y^2+z^2\ge1\)và ta cần chứng minh \(\frac{x^3}{y^2+z^2}+\frac{y^3}{z^2+x^2}+\frac{z^3}{x^2+y^2}\ge\frac{\sqrt{3}}{2}\)(*)

Áp dụng BĐT Bunhiacopxki dạng phân thức, ta được:

\(VT_{\left(^∗\right)}=\frac{x^4}{x\left(y^2+z^2\right)}+\frac{y^4}{y\left(z^2+x^2\right)}+\frac{z^4}{z\left(x^2+y^2\right)}\)\(\ge\frac{\left(x^2+y^2+z^2\right)^2}{x\left(y^2+z^2\right)+y\left(z^2+x^2\right)+z\left(x^2+y^2\right)}\)

Đến đây ta đi chứng minh \(\frac{\left(x^2+y^2+z^2\right)^2}{x\left(y^2+z^2\right)+y\left(z^2+x^2\right)+z\left(x^2+y^2\right)}\ge\frac{\sqrt{3}}{2}\)

\(\Leftrightarrow2\left(x^2+y^2+z^2\right)^2\)\(\ge\sqrt{3}\left[x\left(y^2+z^2\right)+y\left(z^2+x^2\right)+z\left(x^2+y^2\right)\right]\)

Ta có: \(x\left(y^2+z^2\right)=\frac{1}{\sqrt{2}}\sqrt{2x^2\left(y^2+z^2\right)\left(y^2+z^2\right)}\)\(\le\frac{1}{\sqrt{2}}\sqrt{\left(\frac{2x^2+y^2+z^2+y^2+z^2}{3}\right)^3}\)

\(=\frac{2\sqrt{3}}{9}\left(x^2+y^2+z^2\right)\sqrt{x^2+y^2+z^2}\)

Tương tự ta có: \(y\left(z^2+x^2\right)\le\frac{2\sqrt{3}}{9}\left(x^2+y^2+z^2\right)\sqrt{x^2+y^2+z^2}\)

\(z\left(x^2+y^2\right)\le\frac{2\sqrt{3}}{9}\left(x^2+y^2+z^2\right)\sqrt{x^2+y^2+z^2}\)

Cộng theo vế của 3 BĐT trên, ta được: 

\(\text{∑}_{cyc}\left[x\left(y^2+z^2\right)\right]\le\frac{2\sqrt{3}}{3}\left(x^2+y^2+z^2\right)\sqrt{x^2+y^2+z^2}\)

\(\Leftrightarrow\sqrt{3}\text{∑}_{cyc}\left[x\left(y^2+z^2\right)\right]\le2\left(x^2+y^2+z^2\right)\sqrt{x^2+y^2+z^2}\)

Cuối cùng ta cần chứng minh được

\(2\left(x^2+y^2+z^2\right)\sqrt{x^2+y^2+z^2}\le2\left(x^2+y^2+z^2\right)^2\)

\(\Leftrightarrow x^2+y^2+z^2\ge1\)(đúng)

Đẳng thức xảy ra khi \(x=y=z=\frac{1}{\sqrt{3}}\Rightarrow a=b=c=\sqrt{3}\)

15 tháng 5 2018

Mình nhầm, phải là \(\le\frac{1}{3}\)mọi người làm giúp mình với mình cần gấp

1 tháng 8 2020

Theo BĐT Cauchy Schwarz và các biến đổi cơ bản ta dễ có được:
\(\frac{a^2}{\left(2a+b\right)\left(2a+c\right)}=\frac{a^2}{2a\left(a+b+c\right)+2a^2+bc}=\frac{1}{9}\left[\frac{\left(2a+a\right)^2}{2a\left(a+b+c\right)+2a^2+bc}\right]\)

\(\le\frac{1}{9}\left[\frac{4a^2}{2a\left(a+b+c\right)}+\frac{a^2}{2a^2+bc}\right]=\frac{1}{9}\left(\frac{2a}{a+b+c}+\frac{a^2}{2a^2+bc}\right)\)

\(\Rightarrow LHS\le\frac{1}{9}\left(2+\frac{a^2}{2a^2+bc}+\frac{b^2}{2b^2+ca}+\frac{c^2}{2c^2+ab}\right)\)

Tiếp tục theo BĐT Cauchy Schwarz dạng Engel:

\(\frac{a^2}{a^2+2bc}+\frac{b^2}{b^2+2ca}+\frac{c^2}{c^2+2ab}\ge\frac{\left(a+b+c\right)^2}{\left(a+b+c\right)^2}=1\)

Ta thực hiện phép đổi biến thì:

\(\frac{ab}{ab+2c^2}+\frac{bc}{bc+2a^2}+\frac{ca}{ca+2b^2}\ge1\)

Đến đây là phần của bạn

AH
Akai Haruma
Giáo viên
6 tháng 7 2019

Bạn tham khảo lời giải bài 4 link sau:

Câu hỏi của Bonking - Toán lớp 9 | Học trực tuyến

7 tháng 7 2019

Bài giải rất hay, mình cảm ơn bạn nhiều

13 tháng 7 2020

\(\Sigma_{sym}a^4b^4\ge\frac{\left(\Sigma_{sym}a^2b^2\right)^2}{3}\ge\frac{\left(\Sigma_{sym}ab\right)^4}{27}\ge\frac{a^2b^2c^2\left(a+b+c\right)^2}{3}=3a^4b^4c^4\)

13 tháng 7 2020

\(\Sigma\frac{a^5}{bc^2}\ge\frac{\left(a^3+b^3+c^3\right)^2}{abc\left(a+b+c\right)}\ge\frac{\left(a^2+b^2+c^2\right)^4}{abc\left(a+b+c\right)^3}\ge\frac{\left(a+b+c\right)^6\left(a^2+b^2+c^2\right)}{27abc\left(a+b+c\right)^3}\)

\(\ge\frac{\left(3\sqrt[3]{abc}\right)^3\left(a^2+b^2+c^2\right)}{27abc}=a^2+b^2+c^2\)